This page shows a recording of a live class. We're working hard to create our standard, concise explanation videos for the questions in this PrepTest. Thank you for your patience!

This is a strengthening question, as the question stem asks: Which one of the following, if true, most strengthens the argument?

The stimulus starts by telling us that the rate at which children aged 4 and under use safety seats has doubled within the last 8 years. Good to hear! The next sentence begins with the claim that this increased usage of safety seats has prevented child fatalities. This is the conclusion of our argument. The sentence continues by conceding that it is true that child fatalities have risen by 10% during the same period that safety seat usage has increased, but attributes this to a 20% increase in the number of serious accident.

This author’s conclusion is that the safety seats are working—the increase in their usage has prevented fatalities. However, the fact that child fatalities have actually increased during the exact same period is a big problem for this position. The author’s defends his safety seats are working conclusion by pointing to the increase in accidents—it’s not that safety seats aren’t preventing fatalities, its just that there are a lot more opportunities for people to die. The issue with this defense is we have no idea much of the increase in serious automobile accidents involve children. Let’s look at the answer choices and see which answer strengthens the author’s position:

Answer Choice (A) Irrelevant. We are interested in how the number of children aged 4 and under killed could have increased in the same period that safety seat usage increased and prevented fatalities.

Correct Answer Choice (B) This strengthens the argument by making the 20% increase in serious accidents a much better defense of the safety seats are working conclusion. If serious accidents went up 20% while the proportion involving children remained constant, but child fatalities only increased by 10%, it seems that the children are dying at a lower rate than would be expected, which in turn suggests that the safety seats really are having an effect.

Answer Choice (C) If they are in cars the same amount of time then nothing has changed in terms of their chance of being involved in accidents.

Answer Choice (D) We are concerned with children aged 4 and under as a whole, the varying usage of seatbelts between subsets of children aged 4 and under has no impact on our argument

Answer Choice (E) If the adult fatalities increased at the same rate as the child fatalities while the safety seat usage increased, it suggests the doubling of safety seats had no impact on child fatalities.


Comment on this

This page shows a recording of a live class. We're working hard to create our standard, concise explanation videos for the questions in this PrepTest. Thank you for your patience!

We should recognize that this is a strengthening question, as the question stem asks: Which one of the following, if true, most strengthens the argument?

The stimulus begins with a definition of salmonella, which identifies it as a possible cause of intestinal illness. The second sentence tells us that this intestinal illness can be fatal, with an increased likelihood if not identified quickly and treated. So Salmonella can cause intestinal illness which can cause death, especially if you don’t know you have the illness; got it! Following this context about salmonella, we learn about conventional salmonella tests; basically, they suck. They are slow and can miss unusual variants, which since we know you want to quickly identify and treat intestinal illness, is pretty bad. Luckily, there is this new test which identifies whether salmonella is present based of a piece of genetic material which all subsets of salmonella possess. The argument concludes that it would be prudent for public health officials to swap the old conventional tests for the new genetic tests.

A key takeaway from this stimulus you should have is that we need more information about this new test. We know a decent amount about the old one, namely that it’s bad because it is slow and unreliable, but all we really get from the stimulus about the new test is that it is more reliable than the old one. A key detail we are missing is the speed of the test. A necessary assumption of the argument is that the new test isn’t too slow to be a good test for salmonella. As things stand, it is entirely consistent with what we know that the new test is 10 times slower than the old one! While there are always lots of ways to strengthen an argument, the fact that the stimulus mentions the importance of quickly identifying intestinal illness should have us on the lookout for an answer choice that fills the gap in our argument about the new test’s speed. Let’s see what we get:

Answer Choice (A) All this answer choice introduces is a disadvantage of the new test compared to the conventional test. Since our conclusion is that the new test should replace the conventional test, this answer choice actually weakens our argument.

Correct Answer Choice (B) Bingo! This answer gives us another important advantage of the new test over the old test, and fills in our assumption that the new tests aren’t too slow.

Answer Choice (C) Always anchor yourself in the conclusion on strengthening questions. We are looking for reasons why the new test should replace the old test. Our argument is about testing, not treatment.

Answer Choice (D) Ok? How often people get salmonella has nothing to do with relative superiority of the new tests to the old ones. Maybe if, for example, we were told it is hard to mass produce the new tests this answer might do something for us, but as the question is written this answer adds no support.

Answer Choice (E) Cool! We don’t care. Same as C, our argument and conclusion are all about the testing for Salmonella. This answer adds nothing to our argument.


Comment on this

This page shows a recording of a live class. We're working hard to create our standard, concise explanation videos for the questions in this PrepTest. Thank you for your patience!

We should recognize that this is a strengthening question, as the question stem says: Which one of the following, if true, would most help to support the conclusion about the German new-car market?

The stimulus begins by telling us about a recent upsurge in demand for used cars in Germany. It continues by further specifying that this demand is coming from former East Germans who can’t afford to buy new cars and who didn’t have access to cars before the unification of East and West Germany. For real-world context, after World War II Germany was split into the allied-controlled democratic West Germany and Soviet-controlled communist East Germany, which weren’t reunited until the fall of the Berlin Wall. Because there is now more demand for used cars than there are used cars available, the price of used cars has also risen. The next sentence tells us that West Germans, because of this rise in used car value due to East German demand, will be selling their old cars. From all of this information, the author concludes that the new car market will improve as well. This is the conclusion we have been tasked with strengthening.

The first thing we should notice once we finish reading the stimulus is that our conclusion is about new cars while all the support is information about used cars. There is a gap in the argument; specifically, what we need is a reason for why (i) increased East German used car demand, (ii) higher used car prices, and (iii) West Germans selling their used cars, could improve the market for new cars. Let’s take a look at the answer choices:

Answer Choice (A) While this answer choice does mention new cars, it doesn’t help us because it only does so in comparison to West German used car demand, which we know little about. Our conclusion is that the new car market will improve, it is a prediction about a future change. The relative size of one portion of the new car market to one portion of the used car market has no bearing on whether or not the German new car market as a whole will improve.

Answer Choice (B) This does nothing to support our prediction. For one, most European countries could not include Germany, in which case this answer would have zero impact on our prediction. And even if this answer explicitly says Germany was a part of this majority, this just gives a reason why buying a used car might be preferable to buying a new car, and no information suggesting there will be improvement in the new car market.

Answer Choice (C) Who cares? The average number of cars the majority of Germans own across their lifetime has little impact on whether the German new car market will overall improve after the changes in the used car market.

Correct Answer Choice (D) This is exactly what we are looking for. It bridges the gap between one of our premises, the one about an increasing number of West Germans selling their used cars, and our conclusion about the new car market. If West Germans generally buy a new car when they sell a used car, and an increasing number of them are going to be selling their used cars, it seems reasonable that the market for new cars will improve because their will be increased demand from these West Germans selling their old cars.

Answer Choice (E) We aren’t told if these North American cars are used or new, and this a preference of specifically East Germans who we know cannot afford new cars, so this information does nothing for our new car market prediction.


Comment on this

This page shows a recording of a live class. We're working hard to create our standard, concise explanation videos for the questions in this PrepTest. Thank you for your patience!

This is a Sufficient Assumption (SA) question and we know this before the question stem: “… an assumption that would allow the company president’s conclusion to be properly drawn?”

Sufficient assumption questions tend to be very formal. We’re looking for a rule that would validate the conclusion, specifically by bridging the premise and conclusion through the rule. Not only are we extrapolating the rule from our argument, but we’re also using that rule to render the argument “valid.” The way to prephrase our answer choice is by tying our premises and conclusion together into a rule: “If [premise] à then [conclusion].” In this question, this isn’t really the case.

In our first sentence, we’re told that Wilson gives free merch to its top salespeople. We’re not told how the company defines “top salespeople” or how many of them there are.

Next, we’re told that the number of salespeople getting this award has declined a lot over the past 15 years. In response to this, the president of Wilsons says: since our award standard is being a part of the top third of the sales team, we can also conclude that the number of people being passed over for awards has declined.

There is a lot of information in the passage and a couple of assumptions the president is making in his argument. It’s easiest to understand this if we put it into perspective if we have some numbers attached to this.

Fifteen years ago, let’s say in 2000, 100 people were given the award.

Now, in 2015, markedly fewer people got the award. Let’s say 50 people got the award. Based on what the president says, the criteria for awarding the prize is being a part of the top third of the sales force. So, if 50 people are 1/3 of the total sales force, the total sales force would be equal to 150 people. This means that 100 people were NOT awarded. Great!

The only problem is: do we the people who were NOT given an award in 2000? No! How do we know that in 2000, the award criteria is being part of the top 2/3 of the sales force? This would mean that the total sales force was 150, and 50 people were NOT awarded. That would really go against the conclusion.

The president is assuming that because there is a decrease in the awarded, there is a decrease in the NOT awarded. What must we have to guarantee that this is not the case? Membership of one-third OR LESS of the sales force would qualify salespeople for the award. This is our rule.

Answer Choice (A) Hiring policies being the same doesn’t mean anything for the number of people employed OR the number of people passed over for the award. It could be that more people are seeking jobs now than before or vice versa. There are so many ways this could affect the number of people employed and passed over for an award, but no way does it allow us to conclude that the number of people passed over for an award has decreased.

Answer Choice (B) Just because the number of salespeople has increased, that does not mean that the number of people of passed over for an award was higher than 15 years ago. We don’t know the criteria for awards back then, so we can’t draw a conclusion about the people awarded vs not awarded.

Correct Answer Choice (C) It gives us that missing information on the proportion of awardees to the total number of employees, allowing us to conclude that the number of people not awarded has also gone down.

Answer Choice (D) If we plug this back into our stimulus, does this validate our conclusion? No! If sales figures are declining, there could be a number of reasons why, and none of this helps explain why the president is able to conclude that non-awardee numbers have also gone down.

Answer Choice (E) We’re not concerned about calculating sales figures. If this is true, and we have different selection criteria, the number of people passed over for an award could increase or decrease. This does not help validate our conclusion.


2 comments

This page shows a recording of a live class. We're working hard to create our standard, concise explanation videos for the questions in this PrepTest. Thank you for your patience!

This is a sufficient assumption question because of the question stem: “Which one of the following, if established, would help justify…” Note that there are two speakers; we’re specifically focused on validating Oscar’s argument.

Sufficient assumption questions tend to be very formal. We’re looking for a rule that would 100% validate the conclusion, specifically by bridging the premise and conclusion through the rule. Not only are we extrapolating the rule from our argument, but we’re also using that rule to render the argument “valid.” The way to prephrase our answer choice is by tying our premises and conclusion together into a rule: “If [premise] → then [conclusion].”

Oscar’s argument is given first. He’s being accused of plagiarizing the work of Myers but says it is unwarranted. What reason does he give for this? Why should we believe him? He first gives a concession: he used Myer’s work without attribution. Even though this is the case, he didn’t plagiarize because Myer gave him permission for this. So, the conclusion of Oscar’s argument is that the accusation that he’s guilty of plagiarism is unwarranted, and his premise is that he got permission in private correspondence to do use Myer’s work without attribution.

Millie’s argument is basically trying to weaken Oscar’s argument. We know this stimulus is for two different questions, so it’s like that question 6 has something to do with both of the speakers. On the older LSAT, it was very common for the test to give one stimulus for two different questions; this doesn’t happen anymore. We can basically ignore Millie’s claim because we’re trying to validate Oscar’s position. If you do end up reading it, the position Millie takes uncovers a useful implicit assumption that Oscar is making.

Back to the Oscar’s argument: does getting permission from the author to use their work without attribution exclude you from plagiarism? Do we know this from the information in our stimulus? No! This is the implicit assumption that Oscar is making. (This also happens to be the implicit assumption that Millie targets to weaken Oscar’s position). In order for this conclusion to be true, our rule would need to look something like “If I get permission in personal/private correspondence from the author to use their work without attribute, then I’m not guilty of plagiarizing.”

Answer Choice (A) This is one of the ways in which LSAT will routinely trick you – switching the sufficient and necessary conditions. Another way the LSAT will trick you is by putting the sufficient condition later in the sentence and putting the necessary condition first. Notice the “If” later on in the sentence; everything after that is our sufficient condition. With our rule, the direction of the conditional matters. This answer choice has “no right to quote” in the necessary condition, but we want “right to quote” or “not plagiarism.” We can also look at the sufficient condition: we’re looking for getting permission and our answer choice says, “the author hasn’t grant author any right.” This is out.

Answer Choice (B) This is another conditional in which the LSAT has put the sufficient condition later on in the sentence. Rearranged, it reads: “If the quote is more than a few sentences long, the writer of an article must attribute the text.” We can’t even satisfy the sufficient condition to trigger the conditional because we do not know how much Oscar pulled from Myer’s book. In addition, even if we could assume it’s just a few sentences, this would actually weaken the argument. This is also out.

Answer Choice (C) This isn’t correct: we start out we a blanket statement: “Plagiarism is never justified.” But then the answer gives us an exception: “but writers are justified in occasionally quoting without attribution…” Stop here. What would it need to be for this to be the correct answer choice? “When the author gives permission to use their writing without attribution.” That would be great! What does our answer choice say? “If the work has not been published.” This does not apply to our stimulus because we don’t know if it hasn’t/has been published.

Correct Answer Choice (D) The necessary condition is presented first, so if we rearrange the order, we get: “if a writer relinquishes their exclusive right to their work, then another author is entitled to quote freely without attribution.” The “relinquished their exclusive right” is a little dramatic and fancy but saying in private correspondence that someone else can use their work without attribution is relinquishing an exclusive right.

Answer Choice (E) “Quote without attribution what they themselves have written” is the problem here. We’re not talking about what Oscar quotes from his own work; he’s quoting another author. This is out.


Comment on this

Here we have a Method of Reasoning question, which we know from the question stem: “Jermone responds to Ingrid’s claim by…”

After correctly identifying the question type we can use structural analysis to describe the Method of Reasoning used by our speaker. Immediately we should note we have two speakers in our stimulus. That means we need to be on the lookout for two conclusions and two sets of explanations. Our first speaker, Ingrid, lays out a claim; rock has not produced compared to those of the 1940s, because the 1940s records are the ones that continue to be recorded by numerous performers today. Here, Ingrid is assuming a requirement of being a durable song is that it is recorded by numerous performers.

Jerome responds by questioning Ingrid’s assumption. Jerome recognizes that rock songs are only recorded once. However, our second speaker explains this actually reflects the durability of the recordings rather than the lack of popularity of these songs. Jerome challenges Ingrid’s definition of durability to further their own point about the durability of rock music.

Knowing our answer choice will mention the debate surrounding the qualifications of a durable song, we can proceed into answer choice elimination.

Answer Choice (A) We do not have the evidence to suggest that Jermone is intentionally misinterpreting Ingrid’s claim. An answer choice commenting on the intentions of one of the speakers has to follow up with strong confirmatory evidence.

Answer Choice (B) Rather than showing that the claim necessarily leads to a contradiction, Jerome is challenging Ingrid to change the way in which we define durable to begin with.

Answer Choice (C) If our speaker were undermining the truth of Ingrid’s position we would expect Jermone to be challenging the factual content regarding performers in the 1940s. Without questioning the validity of this information we can eliminate answer choice C.

Correct Answer Choice (D) This is exactly what we are looking for. This is the only answer choice that points out how Jermone adjusts the standards for determining what a durable song is.

Answer Choice (E) In order for this answer choice to be correct, we would need to see some indication that Jerome’s argument is based on some sort of character attack. We can eliminate the answer choice Eqwithout this information.


2 comments

Here we have a main conclusion question, which we know from the question stem “The main point of the argument is that...”

The argument opens with a common premise indicator, “since,” so I’m expecting that we’ll first be presented with a reason for the conclusion to be true, and then the conclusion will follow. The premise is that grain companies operating internationally are run with the goal of maximizing profits. Knowing this is supposed to make the following claim more likely to be true: that we can’t rely on these companies to make choices with their money that would reform food distribution worldwide. So, Big Grain can’t reliably solve world hunger because they care too much about making money. The first part of that sentence smells like conclusion to me, as we have a reason to believe it. Let’s see if the rest of the argument supports that, or if this claim ends up supporting a more overarching conclusion. The next sentence includes a concession: sure, sometimes big corporations do things that bring about similar economic change. But hey, it’s just a coincidence that this happens because the right motives weren’t there; these companies that did do economic good didn’t actually care about the world, just about making money. And then we’re given a final reason to agree with the initial claim: maximizing profits usually needs a stable, unchanging economic environment. As terrible as this argument is, there’s no doubt that the conclusion was the second part of the first sentence: we just can’t count on Big Grain to save the world from hunger.

Answer Choice (A) repeats the final sentence of the stimulus almost word-for-word, which we already know isn’t the conclusion because we aren’t presented any reason that this must be true; instead it’s given as support for the actual conclusion. Next!

Answer Choice (B) rephrases the second part of the second sentence, which was also just a premise. The author did say that economic change via big business is motivated by profit, but we’re given no reason to believe this and it serves to support the main conclusion. Still searching...

Answer Choice (C) rephrases the first part of the first sentence, which we immediately deemed a premise due to the “since” and its role in supporting the following half of the sentence (the actual main conclusion). Yes, Big Grain cares about making money, but that’s not our conclusion.

Answer Choice (D) was never stated in the argument. We have reason to think the author here actually would say the opposite, that the world’s current food distribution system does need reform. Why would they even bother writing this argument otherwise? Anyways, it’s definitely not the main conclusion.

Correct Answer Choice (E) is right on the money. Like Big Grain, haha. LSAC made this one pretty easy for us by not even bothering to rephrase the second part of the first sentence of the stimulus for AC (E) beyond removing that step about economic changes, so we are that much more confident that this is our answer!


Comment on this

Here we have a Method of Reasoning question, which we know from the question stem: “Millie uses which one of the following argumentative strategies in contesting Oscar’s position?”

After correctly identifying the question type we can use structural analysis to describe the Method of Reasoning used by our speaker. Immediately we should note we have two speakers in our stimulus. That means we need to be on the lookout for two conclusions and two sets of explanations. Oscar begins by telling us they have been recently accused of plagiarism. Oscar concludes this accusation is unwarranted on the grounds that Myers gave Oscar private permission to use the passages Oscar is accused of plagiarizing.

That makes sense depending on your definition of plagiarism. Our second speaker, Millie, points out the assumption in Oscar’s argument about what it means to plagiarize a piece of writing. Millie concludes that Myers is unable to bestow permission for Oscar to plagiarize because plagiarism itself is a type of lie, and permission to lie does not change the fact that Oscar committed plagiarism.

Millie has undermined Oscar’s position in their response by pointing out Oscar’s conclusion only follows if one changes the meaning of plagiarism. If plagiarism simply means to use another’s writing without permission, then Oscar has not committed plagiarism. As pointed out by Millie, having permission from the author does not change the fact Oscar is using another person’s work for the purpose of misleading the audience.

Knowing our correct answer choice will point out Millie’s debate surrounding this definition, we can proceed into answer choice elimination.

Correct Answer Choice (A) This is exactly what we are looking for! This is the only answer choice that correctly points out the reinterpretation of plagiarism created by the second speaker.

Answer Choice (B) This is not descriptively accurate. In order to invoke evidence to show that Oscar did quote this author we would need to be able to point to an exact line in Millie’s argument referring to the text itself.

Answer Choice (C) This answer choice implies the disagreement between Oscar and Millie concerns if Oscar had permission to use the author’s writing. But our stimuli aren’t concerned with whether the author tried to give Oscar permission. Instead, our speakers are concerned with whether or not the author’s permission qualifies Oscar’s work as falling under the umbrella of plagiarism.

Answer Choice (D) In order for this answer choice to be correct, we would need to be able to identify some sort of “theory of rights” introduced in the stimulus. Without a description of that theory or an explanation as to how we know Oscar aligns with it, we can eliminate this answer choice from consideration.

Answer Choice (E) We are not debating the credibility of either speaker. Instead, we are debating the validity of whether an author can rightfully give another permission to use their work without it qualifying as plagiarism.


Comment on this

This page shows a recording of a live class. We're working hard to create our standard, concise explanation videos for the questions in this PrepTest. Thank you for your patience!

The question stem says “The main point of the argument is that…” Therefore, we know this must be a main conclusion or main point question, asking us to identify the author’s claim that is supported by other claims.

We begin with a claim about the likelihood or probability of something happening. Although it is not a precise claim, we learn that it is “probably” possible, by using human technology, to create a climate on Mars in which living organisms can survive. Right away, I am wondering if this claim will end up supporting another one (making it a premise) or being supported by another claim (making it a conclusion). We don’t quite have enough information yet, so let’s read on.

It looks like we are now being presented with more details as to how this occurrence might happen––it might be hundreds of years before humans could actually live on Mars, even if they had specific technology that would assist them (breathing apparatuses). The second half of this sentence begins with a “but,” usually indicating some sort of contrast or shift in the author’s point. But… we hear about a comparison, well, more than one of the “great temples and cathedrals” also took hundreds of years to build. Side note… huh? How is transforming a currently uninhabitable planet into a place humans could thrive similar to constructing a cathedral? But I digress. Let’s ignore how good or bad this argument seems, as that is not our job to assess in this case––we just need to decipher which is the author’s main conclusion. What role does this contrast play in the argument? Well, at the base level, the author is trying to set up the argument (remember, just a premise and conclusion) that hey, it might take a while until we can inhabit Mars, but didn’t other great feats in history take a while? This claim also introduces a specific piece of human technology, breathing apparatuses, that are implied to assist human life on Mars. So, we are walking away from this second sentence with hope that human technology could allow for life on Mars one day, by way of a comparison and an example. As of now, this second sentence makes the first more likely to be true: now we know, ever so slightly, how “it is probably within the reach of human technology to make the climate of Mars inhabitable.” So, the second sentence supports the first, and is a premise to the best of our knowledge.

Ooh! Wait! We have a third sentence here that builds on the first two, and might be a strong contender for our main conclusion! The claim states that we can prove that research efforts are reasonable if (indicating a sufficient condition) “there is even a chance” of making life on Mars possible. In other words, proving that there is a slight chance of altering Mars’ climate to be inhabitable is enough for us to justify research efforts. How do the first two sentences connect to this? Well, they each work to set up the fact that we may have met the sufficient condition for research to be justified, that we have this slight chance! The first claim tells us that creating conditions of life on Mars is a possibility, and the second sentence partially tells us how it is a possibility, and partially increases the likelihood of it being a possibility by comparison to history. Therefore, the first two claims support, or make more likely to be true, the fact that the sufficient condition in sentence three is met! Let’s read on.

“Besides,” or in addition, we are presented with one potential benefit of the process of making Mars inhabitable. One part of that process, “the intellectual exercise of understanding how the Martian atmosphere might be changed” can assist in understanding a big problem we have on Earth: atmospheric changes due to human activity. So, we have a reason that research into life on Mars would be beneficial, next to some information that sets up the possibility that this goal could truly be achieved. Which one of the author’s claims is being supported overall? This stimulus is similar to other tough main conclusion questions in the sense that these premises are not quite supporting an explicitly stated sentence, but instead we have to put the puzzle pieces together as readers to determine the direction the author is taking the argument. Why would the author have presented us with reasons to believe life on Mars could be possible and list a benefit of researching life on Mars, if they were not trying to let us know that the sufficient condition for justifying research efforts has been met? Remember, it was a pretty weakly worded sufficient condition; it is adequate or enough to establish that “there is even a chance” of making life on Mars possible in order to justify research efforts. The premises tell us that there is this slight chance, so the author’s conclusion must be that research efforts are now justified. Let’s search for this in our AC’s.

Answer Choice (A) This looks like a spot-on rephrase of the first sentence, that technology allows for the possibility of life on Mars. Remember, we thought this might be a supported claim, or the conclusion, after reading the first two sentences, but did the rest of the sentences make this first claim more likely to be true? No, instead, this claim ends up making a later claim more likely to be accepted, that research efforts are justified.

Answer Choice (B) This is a rephrase of the first half of that second sentence, which we determined to be a premise that limits but still sets up the likelihood of life on Mars.

Answer Choice (C) Definitely not our main conclusion. This was an example of other “similar” feats in history, but we can’t make a case that anything else in the argument makes this statement more likely to be true.

Correct Answer Choice (D) There we go! That’s exactly what we are looking for. And, it required some synthesis, or combining different pieces of evidence to support a conclusion, as we have seen other difficult MC questions require of us. After reading this entire stimulus, we are much more likely to accept that research efforts are justified, as the claims support this in some way.

Answer Choice (E) This is a rephrase of that last sentence, listing a potential benefit of pursuing research, which was a piece of support that research efforts are justified.


Comment on this